Wednesday, August 10, 2011

Aptitude test answer

Aptitude Test - Random

AptitudeResult & Statistics
1.By investing in 16% stock at 64, one earns Rs. 1500. The investment made is:
A.
Rs. 5640
B.
Rs. 5760
C.
Rs. 7500
D.
Rs. 9600

Answer: Option B

Explanation:

To earn Rs.50, investment = Rs. 64.
3

To earn Rs. 1500, investment = Rs.64 x3x 1500= Rs. 5760.
50

Learn more problems on : Stocks and Shares

Discuss about this problem : Discuss in Forum

2.A 300 metre long train crosses a platform in 39 seconds while it crosses a signal pole in 18 seconds. What is the length of the platform?
A.
320 m
B.
350 m
C.
650 m
D.
Data inadequate

Answer: Option B

Explanation:

Speed =(300(m/sec =50m/sec.
183

Let the length of the platform be x metres.

Then,(x + 300(=50
393

=> 3(x + 300) = 1950

=> x = 350 m.

Learn more problems on : Problems on Trains

Discuss about this problem : Discuss in Forum

Direction (for Q.Nos. 3 - 4):

Insert the missing number.

3.4, -8, 16, -32, 64, (....)
A.
128
B.
-128
C.
192
D.
-192

Answer: Option B

Explanation:

Each number is the proceeding number multiplied by -2.

So, the required number is -128.

Learn more problems on : Odd Man Out and Series

Discuss about this problem : Discuss in Forum

4.15, 31, 63, 127, 255, (....)
A.
513
B.
511
C.
517
D.
523

Answer: Option B

Explanation:

Each number is double the preceding one plus 1.

So, the next number is (255 x 2) + 1 = 511.

Learn more problems on : Odd Man Out and Series

Discuss about this problem : Discuss in Forum

Direction (for Q.No. 5):

Each of the questions given below consists of a question followed by three statements. You have to study the question and the statements and decide which of the statement(s) is/are necessary to answer the question.

5.

If both the pipes are opened, how many hours will be taken to fill the tank?

I.

The capacity of the tank is 400 litres.

II.

The pipe A fills the tank in 4 hours.

III.

The pipe B fills the tank in 6 hours.

A.
Only I and II
B.
Only II and III
C.
All I, II and III
D.
Any two of the three
E.
Even with all the three statements, answer cannot be given.

Answer: Option B

Explanation:

II. Part of the tank filled by A in 1 hour =1
4

III. Part of the tank filled by B in 1 hour =1
6

(A + B)'s 1 hour's work =1+1=5
4612

A and B will fill the tank in12hrs = 2 hrs 24 min.
5

So, II and III are needed.

Correct answer is (B).

Learn more problems on : Pipes and Cistern

Discuss about this problem : Discuss in Forum

6.The population of a town increased from 1,75,000 to 2,62,500 in a decade. The average percent increase of population per year is:
A.
4.37%
B.
5%
C.
6%
D.
8.75%

Answer: Option B

Explanation:

Increase in 10 years = (262500 - 175000) = 87500.

Increase% =87500x 100% = 50%.
175000

Required average =50% = 5%.
10

Learn more problems on : Percentage

Discuss about this problem : Discuss in Forum

Direction (for Q.No. 7):

Find out the wrong number in the given sequence of numbers.

7.6, 13, 18, 25, 30, 37, 40
A.
25
B.
30
C.
37
D.
40

Answer: Option D

Explanation:

The differences between two successive terms from the beginning are 7, 5, 7, 5, 7, 5.

So, 40 is wrong.

Learn more problems on : Odd Man Out and Series

Discuss about this problem : Discuss in Forum

8.How many times in a day, are the hands of a clock in straight line but opposite in direction?
A.
20
B.
22
C.
24
D.
48

Answer: Option B

Explanation:

The hands of a clock point in opposite directions (in the same straight line) 11 times in every 12 hours. (Because between 5 and 7 they point in opposite directions at 6 o'clcok only).

So, in a day, the hands point in the opposite directions 22 times.

Learn more problems on : Clock

Discuss about this problem : Discuss in Forum

9.A began a business with Rs. 85,000. He was joined afterwards by B with Rs. 42,500. For how much period does B join, if the profits at the end of the year are divided in the ratio of 3 : 1?
A.
4 months
B.
5 months
C.
6 months
D.
8 months

Answer: Option D

Explanation:

Suppose B joined for x months. Then,

Then,85000 x 12=3
42500 x x1

x =85000 x 12= 8.
42500 x 3

So, B joined for 8 months.

Learn more problems on : Partnership

Discuss about this problem : Discuss in Forum

10.Three partners shared the profit in a business in the ratio 5 : 7 : 8. They had partnered for 14 months, 8 months and 7 months respectively. What was the ratio of their investments?
A.
5 : 7 : 8
B.
20 : 49 : 64
C.
38 : 28 : 21
D.
None of these

Answer: Option B

Explanation:

Let their investments be Rs. x for 14 months, Rs. y for 8 months and Rs. z for 7 months respectively.

Then, 14x : 8y : 7z = 5 : 7 : 8.

Now,14x=5 98x = 40y y =49x
8y720

And,14x=5 112x = 35z z =112x =16x.
7z8355

x : y : z = x :49x:16x= 20 : 49 : 64.
205

Learn more problems on : Partnership

Discuss about this problem : Discuss in Forum

Direction (for Q.No. 11):

Each of these questions is followed by three statements. You have to study the question and all the three statements given to decide whether any information provided in the statement(s) is redundant and can be dispensed with while answering the given question.

11.

What will be the ratio between ages of Sam and Albert after 5 years?

I.

Sam's present age is more than Albert's present age by 4 years.

II.

Albert's present age is 20 years.

III.

The ratio of Albert's present age to Sam's present age is 5 : 6.

A.
Any two of I, II and III
B.
II only
C.
III only
D.
I or III only
E.
II or III only

Answer: Option A

Explanation:

Clearly, any two of the given statements will give the answer and in each case, the third is redundant.

Correct answer is (A).

Learn more problems on : Problems on Ages

Discuss about this problem : Discuss in Forum

12.In a bag, there are coins of 25 p, 10 p and 5 p in the ratio of 1 : 2 : 3. If there is Rs. 30 in all, how many 5 p coins are there?
A.
50
B.
100
C.
150
D.
200

Answer: Option C

Explanation:

Let the number of 25 p, 10 p and 5 p coins be x, 2x, 3x respectively.

Then, sum of their values = Rs.25x+10 x 2x+5 x 3x= Rs.60x
100100100100

60x= 30 x =30 x 100= 50.
10060

Hence, the number of 5 p coins = (3 x 50) = 150.

Learn more problems on : Ratio and Proportion

Discuss about this problem : Discuss in Forum

13.The average of 20 numbers is zero. Of them, at the most, how many may be greater than zero?
A.
0
B.
1
C.
10
D.
19

Answer: Option D

Explanation:

Average of 20 numbers = 0.

Sum of 20 numbers (0 x 20) = 0.

It is quite possible that 19 of these numbers may be positive and if their sum is a then 20th number is (-a).

Learn more problems on : Average

Discuss about this problem : Discuss in Forum

Direction (for Q.No. 14):

Each of the questions given below consists of a question followed by three statements. You have to study the question and the statements and decide which of the statement(s) is/are necessary to answer the question.

14.

What is Ravi's present age?

I.

The present age of Ravi is half of that of his father.

II.

After 5 years, the ratio of Ravi's age to that of his father's age will be 6 : 11.

III.

Ravi is 5 years younger than his brother.

A.
I and II only
B.
II and III only
C.
I and III only
D.
All I, II and III
E.
Even with all the three statements answer cannot be determined.

Answer: Option A

Explanation:

I. Let Ravi's present age be x years. Then, his father's present age = 2x years.

II. After 5 years,Ravi's age=6
Father's age11

III. Ravi is younger than his brother.

From I and II, we getx + 5=6. This gives x, the answer.
2x + 511

Thus, I and II together give the answer. Clearly, III is redundant.

Correct answer is (A).

Learn more problems on : Problems on Ages

Discuss about this problem : Discuss in Forum

Direction (for Q.No. 15):

Each of the questions given below consists of a statement and / or a question and two statements numbered I and II given below it. You have to decide whether the data provided in the statement(s) is / are sufficient to answer the given question. Read the both statements and

  • Give answer (A) if the data in Statement I alone are sufficient to answer the question, while the data in Statement II alone are not sufficient to answer the question.
  • Give answer (B) if the data in Statement II alone are sufficient to answer the question, while the data in Statement I alone are not sufficient to answer the question.
  • Give answer (C) if the data either in Statement I or in Statement II alone are sufficient to answer the question.
  • Give answer (D) if the data even in both Statements I and II together are not sufficient to answer the question.
  • Give answer(E) if the data in both Statements I and II together are necessary to answer the question.

15.

The area of a rectangle is equal to the area of right-angles triangle. What is the length of the rectangle?

I.

The base of the triangle is 40 cm.

II.

The height of the triangle is 50 cm.

A.
I alone sufficient while II alone not sufficient to answer
B.
II alone sufficient while I alone not sufficient to answer
C.
Either I or II alone sufficient to answer
D.
Both I and II are not sufficient to answer
E.
Both I and II are necessary to answer

Answer: Option D

Explanation:

Given: Area of rectangle = Area of a right-angles triangle.

l x b =1x B x H
2

I gives, B = 40 cm.

II gives, H = 50 cm.

Thus, to find l, we need b also, which is not given.

Given data is not sufficient to give the answer.

Correct answer is (D).

Learn more problems on : Area

Discuss about this problem : Discuss in Forum

16.
Evaluate :(2.39)2 - (1.61)2
2.39 - 1.61
A.
2
B.
4
C.
6
D.
8

Answer: Option B

Explanation:

Given Expression =a2 - b2=(a + b)(a - b)= (a + b) = (2.39 + 1.61) = 4.
a - b(a - b)

Learn more problems on : Decimal Fraction

Discuss about this problem : Discuss in Forum

17.
.009= .01
?
A.
.0009
B.
.09
C.
.9
D.
9

Answer: Option C

Explanation:

Let.009= .01; Then x =.009=.9= .9
x.011

Learn more problems on : Decimal Fraction

Discuss about this problem : Discuss in Forum

18.Sachin is younger than Rahul by 7 years. If their ages are in the respective ratio of 7 : 9, how old is Sachin?
A.
16 years
B.
18 years
C.
28 years
D.
24.5 years
E.
None of these

Answer: Option D

Explanation:

Let Rahul's age be x years.

Then, Sachin's age = (x - 7) years.

x - 7=7
x9

9x - 63 = 7x

2x = 63

x = 31.5

Hence, Sachin's age =(x - 7) = 24.5 years.

Learn more problems on : Problems on Ages

Discuss about this problem : Discuss in Forum

Direction (for Q.No. 19):

Each of the questions given below consists of a statement and / or a question and two statements numbered I and II given below it. You have to decide whether the data provided in the statement(s) is / are sufficient to answer the given question. Read the both statements and

  • Give answer (A) if the data in Statement I alone are sufficient to answer the question, while the data in Statement II alone are not sufficient to answer the question.
  • Give answer (B) if the data in Statement II alone are sufficient to answer the question, while the data in Statement I alone are not sufficient to answer the question.
  • Give answer (C) if the data either in Statement I or in Statement II alone are sufficient to answer the question.
  • Give answer (D) if the data even in both Statements I and II together are not sufficient to answer the question.
  • Give answer(E) if the data in both Statements I and II together are necessary to answer the question.

19.

How long will it take to empty the tank if both the inlet pipe A and the outlet pipe B are opened simultaneously?

I.

A can fill the tank in 16 minutes.

II.

B can empty the full tank in 8 minutes.

A.
I alone sufficient while II alone not sufficient to answer
B.
II alone sufficient while I alone not sufficient to answer
C.
Either I or II alone sufficient to answer
D.
Both I and II are not sufficient to answer
E.
Both I and II are necessary to answer

Answer: Option E

Explanation:

I. A's 1 minute's filling work =1
16

II. B's 1 minute's filling work =1
8

(A + B)'s 1 minute's emptying work =1-1=1
81616

Tank will be emptied in 16 minutes.

Thus, both I and II are necessary to answer the question.

Correct answer is (E).

Learn more problems on : Pipes and Cistern

Discuss about this problem : Discuss in Forum

20.The banker's discount on Rs. 1600 at 15% per annum is the same as true discount on Rs. 1680 for the same time and at the same rate. The time is:
A.
3 months
B.
4 months
C.
6 months
D.
8 months

Answer: Option B

Explanation:

S.I. on Rs. 1600 = T.D. on Rs. 1680.

Rs. 1600 is the P.W. of Rs. 1680, i.e., Rs. 80 is on Rs. 1600 at 15%.

Time =100 x 80year=1year = 4 months.
1600 x 153

Learn more problems on : Banker's Discount

Discuss about this problem : Discuss in Forum

Marks : 0/20
Total number of questions:20
Number of answered questions:0
Number of unanswered questions:20


Feedback
Quality of the Test:
Difficulty of the Test:
Comments:

0 comments:

Post a Comment

 
Design by Free WordPress Themes | Bloggerized by Lasantha - Premium Blogger Themes | Justin Bieber, Gold Price in India